Apparent angular velocity (inclined orbit)?

In summary, We are trying to determine the angular velocity of a point on the rim of a rotating disk from different observation angles. Although there is a transformation in the observed angular velocity, the actual angular velocity remains constant. This was confirmed by purchasing watches without a circular faceplate where the hands move at constant angular velocity. However, there is a subset of motion where the observed angular velocity can be larger or smaller than the actual angular velocity, depending on the configuration of the observer's angle and the disk's angular velocity.
  • #1
Andy Resnick
Science Advisor
Education Advisor
Insights Author
7,401
3,098
TL;DR Summary
What (changing) angular velocity will I observe?
Here's the problem setup, my student and I are stuck.
Presentation1.jpg


A disk is rotating at constant angular velocity ω, and we are watching a point on the rim, parameterized by the angular position θ, move. Because we are observing the motion from an inclination angle Ψ, we do not always observe the velocity ω but instead a transformed angular velocity that varies with angle θ(t) [and ψ]. This is 'simple geometry' and we figured that we could look up the formula in an astronomy context ('inclined orbits'), but for whatever reason we can't seem to figure it out. Help, please, and thanks in advance!
 
Mathematics news on Phys.org
  • #2
Andy Resnick said:
Summary:: What (changing) angular velocity will I observe?

Here's the problem setup, my student and I are stuck.
View attachment 284903

A disk is rotating at constant angular velocity ω, and we are watching a point on the rim, parameterized by the angular position θ, move. Because we are observing the motion from an inclination angle Ψ, we do not always observe the velocity ω but instead a transformed angular velocity that varies with angle θ(t) [and ψ]. This is 'simple geometry' and we figured that we could look up the formula in an astronomy context ('inclined orbits'), but for whatever reason we can't seem to figure it out. Help, please, and thanks in advance!
Oops- never mind, we figured it out (the angular velocities are the same). Can't delete the thread, sorry!
 
  • #3
Andy Resnick said:
Oops- never mind, we figured it out (the angular velocities are the same). Can't delete the thread, sorry!

I don't think they are the same. If the trajectory projected onto a plane parallel to the disc is ##(x(t),y(t)) = (\cos{\theta(t)}, \sin{\theta(t)})##, where ##\theta(t) = \omega t##, then the trajectory projected onto a tilted plane [hinged along the ##x## axis, say] will be of the form$$(\tilde{x}(t), \tilde{y}(t))= (\cos{\theta(t)}, \lambda \sin{\theta(t)})$$for parameter ##\lambda \in [0,1]##. The observed angle ##\tilde{\theta}(t)## satisfies\begin{align*}
\tan{\tilde{\theta}(t)} = \lambda \tan{\theta(t)} \implies \dfrac{d\tilde{\theta}(t)}{dt} &= \lambda \omega \dfrac{\cos^2{\tilde{\theta}(t)}}{\cos^2{\theta}(t)} \\

\tilde{\omega}(t) &=\lambda \omega \cos^2{\tilde{\theta}(t)} \left(1 + \dfrac{\tan^2{\tilde{\theta}}(t)}{\lambda^2} \right)

\end{align*}It is worth to note that ##\tilde{\omega}(t)## is a function of time.
 
Last edited:
  • Like
Likes Andy Resnick, mfb and FactChecker
  • #4
Andy Resnick said:
Oops- never mind, we figured it out (the angular velocities are the same). Can't delete the thread, sorry!
There is a difference between the angular velocity and the apparent angular velocity to an observer not on the axis of rotation. That is where @ergospherical 's result applies.

CORRECTION: Even to an observer on the axis of rotation, there is a difference between the actual angular velocity and the observed angular velocity (angular rate versus the observer).
 
Last edited:
  • #5
FactChecker said:
CORRECTION: Even to an observer on the axis of rotation, there is a difference between the actual angular velocity and the observed angular velocity (angular rate versus the observer).

A camera on the axis, corresponding to orthogonal projection onto a plane with ##\lambda = 1##, results in ##\tilde{\omega}(t) = \omega## so there will be no difference in this case.
 
  • Like
Likes FactChecker
  • #6
ergospherical said:
A camera on the axis, corresponding to orthogonal projection onto a plane with ##\lambda = 1##, results in ##\tilde{\omega}(t) = \omega## so there will be no difference in this case.
I agree. It is not clear to me exactly what the OP was asking about.
1) The true angular acceleration, not considering an external observer.
2) The orthogonal projection from an observer.
3) The azimuth/elevation from the viewpoint of an observer.
 
  • #7
I am certain it is not proposition (1) because the "true" angular velocity of the disc, with respect to a set of basis vectors ##\mathcal{B} = \{ \mathbf{e}_1, \mathbf{e}_2, \mathbf{e}_3 \}## fixed in the lab, is the vector ##\boldsymbol{\omega}## such that any vector ##\mathbf{a}## rigidly embedded in the disc satisfies* ##\dfrac{d\mathbf{a}}{dt} \bigg{)}_{\mathcal{B}} = \boldsymbol{\omega} \times \mathbf{a}##. As such it is independent of any "observer".

Proposition (2) is what I had in mind. The orthogonal projection onto a plane represents what an observer actually sees (after their brain has "flipped" the real image formed on the retina).

*N.B. ##\dfrac{d\mathbf{a}}{dt} \bigg{)}_{\mathcal{B}} := \dfrac{da_1}{dt} \mathbf{e}_1 + \dfrac{da_2}{dt} \mathbf{e}_2 + \dfrac{da_3}{dt} \mathbf{e}_3##
 
  • #8
ergospherical said:
I am certain it is not proposition (1) because the "true" angular velocity of the disc, with respect to a set of basis vectors ##\mathcal{B} = \{ \mathbf{e}_1, \mathbf{e}_2, \mathbf{e}_3 \}## fixed in the lab, is the vector ##\boldsymbol{\omega}## such that any vector ##\mathbf{a}## rigidly embedded in the disc satisfies* ##\dfrac{d\mathbf{a}}{dt} \bigg{)}_{\mathcal{B}} = \boldsymbol{\omega} \times \mathbf{a}##. As such it is independent of any "observer".
Yes, but that is what he concluded was the answer in post #2. So it is not clear to me if that is what he had in mind.
 
Last edited:
  • #9
ergospherical said:
I don't think they are the same.

FactChecker said:
Yes, but that is what he concluded was the answer in post #2. So it is not clear to me if that is what he had in mind.

Our tentative logic for the angular velocities being the same goes like this:

The amount of time for the point to execute 1 complete orbit (or a half-orbit, or a quarter-orbit...) is the same for observing at any angle ψ. Since the angular velocity is never observed to be faster than the 'proper' angular velocity, the angular velocity must be the same always.

Does that make sense?

Edit: another fact supporting our argument is that you can purchase watches without a circular faceplate- the numbers are not equispaced but the minute and hour hands both move at constant angular velocity.
 
Last edited:
  • #10
Andy Resnick said:
The amount of time for the point to execute 1 complete orbit (or a half-orbit, or a quarter-orbit...) is the same for observing at any angle ψ. Since the angular velocity is never observed to be faster than the 'proper' angular velocity, the angular velocity must be the same always.

The error is that the observed angular velocity ##\tilde{\omega}## is in fact larger than ##\omega## for a subset of the motion (which depends on ##\lambda## and ##\omega##).

With the configuration as in my post #3, ##\tilde{\omega}## exhibits maxima with ##\tilde{\omega} > \omega## at ##\tilde{\theta} = \left(n+\dfrac{1}{2} \right)\pi## and exhibits minima with ##\tilde{\omega} < \omega## at ##\tilde{\theta} = m\pi##, for integral values of ##m## and ##n##.
 
  • #11
Andy Resnick said:
Our tentative logic for the angular velocities being the same goes like this:

The amount of time for the point to execute 1 complete orbit (or a half-orbit, or a quarter-orbit...) is the same for observing at any angle ψ. Since the angular velocity is never observed to be faster than the 'proper' angular velocity, the angular velocity must be the same always.

Does that make sense?
That logic makes sense if we assume that we are talking about the average and the range is always the same.
 
  • #12
Andy Resnick said:
Edit: another fact supporting our argument is that you can purchase watches without a circular faceplate- the numbers are not equispaced in angle but the minute and hour hands both move at constant angular velocity.

Just to comment on this edit; the situation you describe is quite impossible. Either the numbers are equally spaced in angle, or the minute and hour hands do not rotate at a constant rate.
 

1. What is apparent angular velocity in an inclined orbit?

Apparent angular velocity refers to the rate at which an object appears to move along its orbital path as observed from a fixed point on Earth. In an inclined orbit, the object's path is not aligned with the Earth's equator, resulting in a varying apparent angular velocity.

2. How is apparent angular velocity calculated?

Apparent angular velocity can be calculated using the formula: ω = v / r, where ω is the angular velocity, v is the object's linear velocity, and r is the distance from the object to the center of rotation.

3. What factors affect the apparent angular velocity in an inclined orbit?

The apparent angular velocity in an inclined orbit is affected by the object's orbital speed, the inclination angle of the orbit, and the observer's location on Earth.

4. How does apparent angular velocity differ from true angular velocity?

Apparent angular velocity is the rate at which an object appears to move, while true angular velocity is the actual rate at which the object is moving along its orbital path. In an inclined orbit, these two velocities may differ due to the tilt of the orbit.

5. Why is apparent angular velocity important in astronomy?

Apparent angular velocity is important in astronomy because it allows scientists to accurately track the movement of objects in the sky. By understanding the apparent angular velocity of celestial bodies, scientists can make predictions about their future positions and study their orbits in more detail.

Similar threads

  • Introductory Physics Homework Help
Replies
9
Views
968
Replies
12
Views
4K
  • Introductory Physics Homework Help
Replies
14
Views
2K
  • Introductory Physics Homework Help
Replies
12
Views
4K
  • Introductory Physics Homework Help
Replies
4
Views
1K
  • Introductory Physics Homework Help
Replies
8
Views
2K
  • Engineering and Comp Sci Homework Help
Replies
5
Views
3K
  • Introductory Physics Homework Help
Replies
3
Views
1K
  • Introductory Physics Homework Help
Replies
2
Views
877
  • Beyond the Standard Models
Replies
6
Views
371
Back
Top